¿Por qué la fuerza débil distingue entre zurdos y diestros?

Me pregunto por qué la interacción débil solo afecta a las partículas levógiras (y antipartículas levógiras).

Antes de que alguien diga "porque así es la naturaleza" :-), permítanme explicar lo que encuentro que necesita una explicación:

En el límite de los fermiones sin masa, la quiralidad se convierte en helicidad. ( S pags ^ ) . Ahora bien, la helicidad es una propiedad del estado de movimiento de un objeto en el espacio. Es bastante obvio para mí cómo la simetría interna S tu ( 2 ) × tu ( 1 ) "sabría" al respecto y sería capaz de distinguir los dos estados de movimiento de helicidad diferentes.

En un nivel más técnico, IIRC, los espinores diestros y zurdos se distinguen por sus propiedades de transformación bajo ciertas transformaciones de espacio-tiempo, y se definen independientemente de cualquier simetría interna. Si queremos obtener el comportamiento de violación de paridad / VA observado, tenemos que conectar un factor de ( 1 γ 5 ) explícitamente en el lagrangiano.

¿Hay alguna razón por la que esto tiene que ser así? ¿Por qué no hay fuerza de acoplamiento solo para partículas dextrógiras? ¿Por qué no hay ( 1 + γ 5 ) ¿término? Tal vez exista en un nivel más fundamental, pero ¿esta simetría está rota?

Er... "por qué" a veces es difícil de responder cuando se pregunta sobre un simple hecho de observación. Al menos hasta que tengas una teoría más fundamental.
Bueno, por supuesto :-). Hay algunas observaciones que uno simplemente toma como hechos de la naturaleza, pero a veces uno puede tratar de buscar una comprensión más profunda tratando de explicarlas. El hecho de que todos los planetas estén (casi) en el mismo plano puede darse por sentado, o puede decirnos algo sobre la formación del sistema solar. O simplemente podemos notar que las partículas elementales tienen masas y ponerlas en nuestras ecuaciones (y vivir con una teoría inconsistente, pero a la naturaleza no le importa necesariamente la elegancia matemática), o podemos razonar sobre EWSB y el mecanismo de Higgs.
@jdm: actualmente no hay una comprensión más profunda del desglose de la conservación de la paridad en la interacción débil. Si lo descubre, bien puede recibir un Premio Nobel.
@Karsus Ren: Es más fácil decirlo que hacerlo :-)
Ver también: Richad Fineman analiza "¿Por qué?" preguntas, en un nivel fundamental, usando el ejemplo de "¿Por qué los imanes se repelen?" (Vale la pena verlo por la brillantez de la explicación) Richard Feynman. Por qué. Entrevista de la BBC, 7:32 min en youtu.be/36GT2zI8lVA
Aquí hay algunas razones posibles. Tal vez este universo no sea real y solo sea simulado por otro universo cuyas leyes son invariantes bajo paridad como el juego de la vida de Conway. Tal vez las leyes observadas son un conjunto de leyes más estricto que las leyes fundamentales que son invariantes bajo paridad y la varianza de paridad ocurre por una razón similar a la homoquiralidad de las moléculas biológicas en la Tierra.

Respuestas (4)

Creo que estás invirtiendo la lógica de la quiralidad y la helicidad en el límite sin masa. La quiralidad define en qué representación del grupo de Lorentz se transforman los espinores de Weyl. No se "convierte" en helicidad, la helicidad "se convierte" en quiralidad en el límite sin masa. Es decir, la quiralidad es lo que es, y define una representación de un grupo y eso no puede cambiar. Esta otra cosa que hemos definido llamada helicidad resulta ser lo mismo en un límite particular.

Ahora, una vez que toma el límite sin masa, los fermiones de Weyl viajan a la velocidad de la luz, ya no puede aumentar a un marco que cambia la helicidad. Creo que es mejor pensar en un término de masa de fermión como una interacción en este caso y recordar que el término de masa de un fermión de Dirac es un montón de tipos de Weyl diestros y zurdos chocando entre sí en el camino. Por el contrario, si desea hablar sobre un fermión de Dirac masivo completo que viaja menos de c y puede aumentar para cambiar la helicidad, pero ese fermión de Dirac completo no es lo que lleva una carga débil, solo una 'parte' lo es.

Vea esta publicación de blog sobre helicidad y quiralidad.

En cuanto a la ruptura de la simetría izquierda-derecha , la gente ciertamente ha construido modelos en este sentido, pero no creo que hayan funcionado.

¿Responde esto a tu pregunta?

Bueno, sé que la quiralidad es una propiedad intrínseca y no cambia, mientras que la helicidad depende del marco. Podría haber dicho que se vuelven iguales, o que hay una correspondencia 1:1 entre ellos, etc.
Gracias por los enlaces. El primero es muy bonito. Y de hecho, su último párrafo sobre las teorías simétricas izquierda-derecha era lo que estaba buscando.

La explicación es simple --- todas las partículas que podemos ver son quirales, tienen solo una mano, porque si tuvieran ambas manos, podrían tener una masa, y genéricamente, esa masa sería del orden de magnitud de la masa de Planck. Vivimos en escalas de energía que son diminutas en comparación con la masa de Planck, por lo que solo podemos ver cosas sin masa, por lo que solo vemos fermiones quirales (y bosones de calibre).

La pregunta correcta entonces es al revés, si todo es quiral, ¿por qué entonces las interacciones electromagnética y fuerte no violan la paridad?

Esto se debe a que el mecanismo de Higgs asocia las quiralidades en partículas masivas de Dirac a energías más bajas, y solo los bosones W, Z saben que, para empezar, eran quirales. A bajas energías, obtienes paridad y conjugación de carga como simetrías accidentales, porque estas son simetrías del Lagrangiano de Dirac de baja energía acoplado al fotón y gluones restantes.

En cuanto a los neutrinos, un neutrino quiral puede tener una masa de Majorana teniendo solo una quiralidad, y esto es ciertamente lo que sucede en la naturaleza, ya que este esquema predice la masa correctamente, y esta masa es absurdamente pequeña.

Ron, por ejemplo, si tanto el fermión izquierdo como el derecho viven en la misma representación de SU(2), ¿por qué tendrían una masa del orden de la escala de Planck?
@Newman: Porque habría un parámetro en el Lagrangiano con masa de dimensión que les daría masa, y ¿por qué debería ajustarse a algo muy pequeño?
pero no habría simetría quiral en el límite de masas, por lo que se cumpliría esa condición de naturalidad_
@Newman: No estoy de acuerdo con que esta sea la declaración correcta de la condición de naturalidad. La "simetría quiral" no es la condición correcta. "Invariancia de calibre que prohíbe los términos de masa" es la condición correcta. Cuando escribes artificialmente los 2 espinores como 2 componentes de los 4 espinores, esto parece una simetría quiral, porque esto es lo que desacopla los otros dos componentes, pero esto es un truco de notación. Las cosas son 2-spinores en realidad. ¿Por qué debería uno imponer una simetría quiral global? Estoy de acuerdo en que si lo impone, puede hacer una afinación natural m = 0, lo que significa que los términos de orden superior no lo romperán.

No hay nada a priori que diga que tiene que ser así y (creo) las oscilaciones de neutrinos ahora han demostrado que tienen una masa que implica que existen tanto neutrinos diestros como zurdos. Dicho esto, el lagrangiano todavía está seriamente sesgado hacia la interacción de la mano izquierda. Así que todavía es muy levógiro en cierto sentido, pero no hay nada que diga que un acoplamiento a las partículas levógiras está descartado.

Simplemente no vemos un acoplamiento fuerte experimentalmente.

Busque la cuestión (aún no resuelta) de Dirac versus la naturaleza de Majorana de los neutrinos. La masa distinta de cero es necesaria pero no suficiente para requerir que ambas manos estén presentes.
@dmckee Buen punto. Ha pasado un tiempo desde que estuve activo en física de partículas. Tengo que hacer una actualización sobre eso.
@unclejamil: Es (teóricamente) casi seguro que los neutrinos no tienen un componente diestro y obtienen su masa al mezclarse con la antipartícula. Esto está abierto experimentalmente, pero uno haría cualquier apuesta teóricamente contra las masas de Dirac. Aunque dmckee te corrigió, lo que escribiste es lo contrario de lo que es más probable --- lo más probable es que el neutrino sea una partícula masiva con dos helicidades que es su propia antipartícula a velocidades no relativistas.

Esta respuesta es similar a la de Ron Maimon, pero tal vez sea útil.

En resumen, la fuerza débil no viola la paridad; sólo los fermiones lo hacen.

No hay fermiones de Dirac en el modelo estándar. No hay proyecciones quirales en el Lagrangiano porque no hay nada sobre lo que puedan actuar. No hay matrices gamma en absoluto, solo espinores de Weyl de dos componentes y matrices de Pauli.

A baja temperatura, hay algunas combinaciones de campos de Weyl que se comportan para muchos propósitos prácticos como los fermiones de Dirac. Si ve un Lagrangiano de modelo estándar con espinores de Dirac, es uno en el que los campos de Weyl se han emparejado "en previsión de" que esta descripción aproximada de baja energía sea útil. Los operadores de proyección quiral están ahí para dividirlos nuevamente cuando sea necesario. Técnicamente, nunca deberían haberse unido en primer lugar. La fuerza débil no está proyectando; es un invento humano.

El comportamiento similar al de Dirac proviene de los acoplamientos de Yukawa que involucran el campo de Higgs. El campo de Higgs tiene carga SU(2) y U(1) pero no carga SU(3) y, como resultado, los únicos acoplamientos Yukawa invariantes de calibre posibles son entre campos de fermiones cuyas cargas SU(3) coinciden y cuyas SU(2) coinciden. ) y los cargos U(1) no. Las fuerzas de calibre no "saben" nada de esto; se acoplan a lo que sea que tenga carga, pero debido a la forma en que los campos cuasi-Dirac se acoplan entre sí, sus mitades tienen cargas SU(2) y U(1) no coincidentes, pero cargas SU(3) iguales.

Como se menciona en la respuesta de Ron Maimon, una razón plausible por la que "tiene que ser así" es que no hay nada que impida un acoplamiento directo entre los campos de fermiones con simetría especular, es decir, un término de masa real, y ninguna razón conocida por la cual la masa resultante no sería comparable a la masa de Planck. Así que probablemente no veríamos campos de fermiones simétricos incluso si existieran. Por qué existen los fermiones asimétricos es una pregunta abierta. Creo que muchos enfoques prometedores de la gravedad cuántica han tenido que abandonarse porque no pueden adaptarse a los fermiones quirales.